Grow-Again Confusion

ehoyaehoya Member
Can anybody help me understand question 15 from PT9, S4 included in the Strengthening Questions Problem Set 8? I felt completely blindsided by how answer C (The rebate, if offered, would not attract purchasers who otherwise might not use Grow-Again) strengthens the connection between the premises and the conclusion. Am I reading this right?

P1: Five drops daily [of a treatment for reversing male hereditary baldness] is the recommended dose
P2: exceeding this quantity [of five drops daily] does not increase the product's effectiveness
SC: *Therefore,* offering a manufacturer's rebate will not increase sales
C: *consequently,* offering a rebate would not be profitable

My first bit of confusion was over how the author jumped from the quantity and effectiveness of the dosage to a rebate increasing (or failing to increase) sales. Are we supposed to assume that had an increase in the dosage improved its effectiveness, then a rebate would have increased the sales? If not, what's the link between A (the premises) and B (the sub conclusion)?

My second area of puzzlement was over how the correct answer (according to the breakdown at the bottom of the Problem Set), which talks about purchasers who otherwise wouldn't use Grow-Again relates to a discussion of dosages, effectiveness, and profitability of rebates. Are the purchasers we're talking about men suffering from hereditary baldness who have chosen other products (in which case perhaps improving the effectiveness, or at least the perception of the effectiveness, might make a rebate more profitable)? Or are we talking about men who aren't suffering from hereditary baldness or women or cute bald babies or pink flamingoes who, obviously, would have no interest in the product to begin with, regardless of any sort of manufacturer's rebate?

In any case, what does any of this have to do with the information given as the premises? Strengthening questions are supposed to beef up the link between the premises and the conclusion, right? So shouldn't a correct answer for a strengthening question have *something* to do with both items? I was so confused by the stimulus to begin with that I chose answer D (Grow-Again is a product whose per-unit manufacturing cost does not fall significantly when the product is produced in large quantities) because it was the only thing I saw that seemed to give some sort of connection (albeit very, very tenuous) between the information about the quantity of the product mentioned in the premises and the cost/profitability mentioned in the sub-conclusion and conclusion.

Help!

Comments

  • c.janson35c.janson35 Free Trial Inactive Sage Inactive ⭐
    edited July 2015 2398 karma
    @ehoya said:
    My first bit of confusion was over how the author jumped from the quantity and effectiveness of the dosage to a rebate increasing (or failing to increase) sales. Are we supposed to assume that had an increase in the dosage improved its effectiveness, then a rebate would have increased the sales? If not, what's the link between A (the premises) and B (the sub conclusion)?
    This is a good thing to be confused about because it is completely unsupported! You, like me, probably read the part about rebates and thought, "ummmmm... excuse me?". Whenever there is a big jump between the premises and conclusion, usually represented in terms of an out-of-nowhere claim/variable that appears for the first time in the conclusion, it is a strong indication that the author is making some serious assumptions. Here, the author is assuming that a decrease in price via rebate will not lead to an increase in sales, which would make the rebate unprofitable (because Grow-Again would just be selling the same product to existing customers for cheaper).

    But what if the reduced price made the product more accessible and attractive to a more men? Then it seems like the rebate would increase sales by increasing their customer base. This is a serious oversight on behalf of the author--he completely "fails to consider this." Therefore, knowing that the rebate would not do this would eliminate this one weakness, which would strengthen the argument in a sort of addition-by-subtraction way. This is exactly what C does, and is why it is the correct answer: it eliminates a potential weakness of the argument.

    Moreover, you've already identified intuitively the gap between the conclusion and premises (that "huhhhhhh?" feeling), so it is more than likely that the correct answer will deal with that gap. Only C discusses the idea of a rebate, so this is another reason to hone in on it.
Sign In or Register to comment.